Last visit was: 23 Apr 2024, 14:45 It is currently 23 Apr 2024, 14:45

Close
GMAT Club Daily Prep
Thank you for using the timer - this advanced tool can estimate your performance and suggest more practice questions. We have subscribed you to Daily Prep Questions via email.

Customized
for You

we will pick new questions that match your level based on your Timer History

Track
Your Progress

every week, we’ll send you an estimated GMAT score based on your performance

Practice
Pays

we will pick new questions that match your level based on your Timer History
Not interested in getting valuable practice questions and articles delivered to your email? No problem, unsubscribe here.
Close
Request Expert Reply
Confirm Cancel
SORT BY:
Date
Tags:
Show Tags
Hide Tags
Intern
Intern
Joined: 22 Sep 2018
Posts: 49
Own Kudos [?]: 47 [0]
Given Kudos: 44
Send PM
GMAT Club Verbal Expert
Joined: 13 Aug 2009
Status: GMAT/GRE/LSAT tutors
Posts: 6917
Own Kudos [?]: 63649 [0]
Given Kudos: 1773
Location: United States (CO)
GMAT 1: 780 Q51 V46
GMAT 2: 800 Q51 V51
GRE 1: Q170 V170

GRE 2: Q170 V170
Send PM
Manager
Manager
Joined: 23 Feb 2020
Posts: 134
Own Kudos [?]: 87 [0]
Given Kudos: 297
Location: Nepal
GMAT 1: 650 Q44 V35
Send PM
GMAT Club Verbal Expert
Joined: 13 Aug 2009
Status: GMAT/GRE/LSAT tutors
Posts: 6917
Own Kudos [?]: 63649 [1]
Given Kudos: 1773
Location: United States (CO)
GMAT 1: 780 Q51 V46
GMAT 2: 800 Q51 V51
GRE 1: Q170 V170

GRE 2: Q170 V170
Send PM
Re: In two months, the legal minimum wage in the country of Kirlandia will [#permalink]
1
Bookmarks
Expert Reply
Mck2023 wrote:
Hi GMATNinja,
How do we determine that this part: 'the resulting rise in wages will drive the inflation rate up', is not a conclusion. I thought that this (the above part) and 'no impact wages' both are conclusion, and searched for the answer choice that will weaken any of these two. I think there is some fault in my approach, but do not know what exactly. Please shed some lights on how to find the exact conclusion in this type of question arguments? Thank you!!!

When searching for a conclusion, you want to determine why the author wrote the passage. What is he/she trying to convince you, the reader, to be true?

The two statements that you've highlighted go against one another. The first states the view of some opponents of the increased minimum wage, who think that "the resulting rise in wages will drive the inflation rate up." In other words, these opponents think that the increased minimum wage will have a bad effect.

Then the author argues against this claim, saying that in fact, if the minimum wage is increased, "the impact on wages will probably be negligible." In other words, wages won't be impacted that much, so we don't need to worry about the bad effect mentioned by the opponents.

So, overall, why did the author write this passage? He/she is trying to convince you that the increase in minimum wage won't have a big negative impact on inflation. This is expressed in the line "its impact on wages will probably be negligible." The previous line serves as a counterpoint to this conclusion. To answer the question, we just need to weaken the author's argument that the impact of increasing the minimum wage will probably be negligible.

I hope that helps!
Manager
Manager
Joined: 27 Mar 2016
Posts: 191
Own Kudos [?]: 5 [0]
Given Kudos: 101
Send PM
Re: In two months, the legal minimum wage in the country of Kirlandia will [#permalink]
In below lines from argument. ''its impact on wages will be negligible'' whose impact? the rise in inflation's impact ??

''Opponents of this increase have argued that the resulting rise in wages will drive the inflation rate up. In fact its impact on wages will probably be negligible ''
Target Test Prep Representative
Joined: 19 Jul 2022
Posts: 430
Own Kudos [?]: 507 [1]
Given Kudos: 1
GMAT 1: 800 Q51 V51
Send PM
Re: In two months, the legal minimum wage in the country of Kirlandia will [#permalink]
1
Kudos
Expert Reply
himanshu0123 wrote:
In below lines from argument. ''its impact on wages will be negligible'' whose impact? the rise in inflation's impact ??

''Opponents of this increase have argued that the resulting rise in wages will drive the inflation rate up. In fact its impact on wages will probably be negligible ''


"Its impact on wages" refers to the impact of instituting the new, higher minimum wage.

In any case—You're right that this argument is very badly written. There's no way this problem is from GMAC.
In addition to the sloppy writing that you pointed out, there's a bigger problem: The conclusion of this argument isn't even there! (This argument is made to support the conclusion that the inflation rate is unlikely to increase.)
Manager
Manager
Joined: 13 Jun 2021
Posts: 64
Own Kudos [?]: 42 [0]
Given Kudos: 284
Location: India
Concentration: Finance, Technology
Schools: ISB '23 (II)
GMAT 1: 660 Q49 V32
GPA: 4
Send PM
Re: In two months, the legal minimum wage in the country of Kirlandia will [#permalink]
While I agree that C is the best choice that weakens the argument presented here, I am looking for correction on my approach as I was confused between option C and D, and this confusion made me waste precious time during the mock.

In Option D, From 'workers earn wages that tend to be much higher than the minimum wage' I inferred that the difference between the minimum wage and workers wage (from particular sectors) needs to be high, i.e., Worker's Wage - Minimum Wage = V. High
So if minimum wage increases, the worker's wage needs to be increased to maintain the greatest growth

Let me know where my analysis needs correction.
Director
Director
Joined: 17 Aug 2009
Posts: 623
Own Kudos [?]: 31 [0]
Given Kudos: 21
Send PM
Re: In two months, the legal minimum wage in the country of Kirlandia will [#permalink]
In two months, the legal minimum wage in the country of Kirlandia will increase from five Kirlandic dollars (KD5.00) per hour to KD5.50 per hour. Opponents of this increase have argued that the resulting rise in wages will drive the inflation rate up. In fact its impact on wages will probably be negligible, since only a very small proportion of all Kirlandic workers are currently receiving less than KD5.50 per hour.

Conclusion - The impact on the wages will be negligible (we are not talking about the impact on inflation - they are interrelated but the conclusion is specifically about the impact on WAGES and not on inflation)

Which of the following, if true, most seriously weakens the argument?


(A) Most people in Kirlandia who are currently earning the minimum wage have been employed at their current jobs for less than a year - less than a year or more than a year, doesn't matter. Out of scope

(B) Some firms in Kirlandia have paid workers considerably less than KD5.00 per hour, in violation of Kirlandic employment regulations. - Some can be at least 2 of the already very small population. At best, it is a distortion or half-truth. First, we don't know who didn't follow the rules earlier will follow now, and secondly, even if they follow the segment is very small, the conclusion still stays intact.

(C) Many businesses hire trainees at or near the minimum wage but must reward trained workers by keeping their pay levels above the pay level of trainees. - This weakens. In a Weaken question, expect a piece of outside information. We are told about trainees (a group that wasn't mentioned in the argument - but this is actually ok as we expect a new piece of information in a weaken question, information that falls within the scope of the argument), if intern salary goes to 5.5 then for a large percentage of people the wages further needs to increase from 5.5, leading to huge impact on the WAGES.

(D) The greatest growth in Kirlandia's economy in recent years has been in those sectors where workers earn wages that tend to be much higher than the minimum wage. - This option talks about the people who earn much higher than the minimum wage - the scope is concerned about the minimum wage - at best this is out of scope

(E) The current minimum wage is insufficient for a worker holding only one job to earn enough to support a family, even when working full time at that job. - out of scope, which is limited to the impact of the increase of minimum wage on wages.
GMAT Club Bot
Re: In two months, the legal minimum wage in the country of Kirlandia will [#permalink]
   1   2 
Moderators:
GMAT Club Verbal Expert
6917 posts
GMAT Club Verbal Expert
238 posts
CR Forum Moderator
832 posts

Powered by phpBB © phpBB Group | Emoji artwork provided by EmojiOne